Tools
Mathematik: Ein schwieriges Problem auf der IMO
Released by matroid on So. 08. Dezember 2019 08:36:17 [Statistics] [Comments]
Written by trunx - 2317 x read [Outline] Printable version Printer-friendly version -  Choose language   
Mathematik

\(\begingroup\)\(\usepackage{setspace}\) Auf der Wikipediaseite "Internationale Mathematik-Olympiade" werden die zwei schwersten Probleme genannt, die je auf einer IMO gestellt worden sind. Beide Aufgaben konnten nur von 11 Schülern gelöst werden, einmal (1986) bei insgesamt 210, das zweite Mal (1988) bei insgesamt 268 Teilnehmern. Während für die erste dieser Aufgaben auch eine Lösung verlinkt wurde, habe ich für die zweite Aufgabe keine Lösung im Internet gefunden (aber auch nicht wirklich intensiv danach gesucht). Da es zudem hieß, dass weder die Mitglieder des Aufgabenausschusses noch von ihnen beauftragte Mathematiker des entsprechenden Fachgebietes (Zahlentheorie) die Aufgabe in 6h lösen konnten, war bei mir das Interesse geweckt. Die Aufgabe lautete (siehe hier): Let \(a\) and \(b\) positive integers such that \(ab+1\) divides \(a^2 +b^2\). Show that \[\frac{a^2 +b^2}{ab+1}\] is the square of an integer. (dt. lt. wikipedia: Sind \(a\) und \(b\) natürliche Zahlen, sodass \[c=\frac{a^2 +b^2}{ab+1}\] ebenfalls eine natürliche Zahl ist, ist c sogar eine Quadratzahl.) Ich habe deutlich mehr als 6h für die Lösung gebraucht, aber es hat Spass gemacht. Daher, wer es selbst probieren will, macht jetzt besser den PC aus und rechnet! Nachtrag: Die nachgelieferte Zuendeführung des angekündigten Beweises findet sich im nächsten Abschnitt in blauer Schrift.

Beweis

Die entscheidende Beweisidee ist folgende: Wäre obiges c als Lösung eine nicht näher spezifizierte Zahl q, dann gibt es natürlich für a und b jeweils q-adische Darstellungen. Diese seien \[a=\sum \limits_{i=0}^{n} a_i q^i\] \[b=\sum \limits_{j=0}^{m} b_j q^j\] Wir setzen nun in \(c\cdot (ab+1)=q\cdot (ab+1)=a^2 +b^2\) diese beiden Darstellungen ein und machen einen Koeffizientenvergleich zwischen den so entstehenden Polynomen in q. Man erhält \((1+a_0 b_0 )q +(a_0 b_1 +a_1 b_0 )q^2 +(a_0 b_2 +a_1 b_1 +a_2 b_0)q^3 + ... =a_0^2 +b_0^2 +2(a_0 a_1 +b_0 b_1 )q+(2a_0 a_2 +a_1^2 +2b_0 b_2 +b_1^2 )q^2 +...\) Daraus erhält man zunächst \(a_0^2 +b_0^2 =0\) und damit \(a_0 =b_0 =0\). Aber schon die Koeffizienten von q können nicht gleich werden, d.h. um die gesamte Gleichung doch noch erfüllen zu können, müsste \(q=0\) gesetzt werden, es wäre also a = b = c = 0; c kann also nicht einfach irgendeine natürliche Zahl sein. Lt. Aufgabenstellung ist sie das auch nicht, wir setzen also \(c=q^2\) an, die q-adischen Darstellungen von \(a\) und \(b\) bleiben. Diese eingesetzt in nunmehr \(q^2 (ab+1)=a^2 +b^2\) ergibt: \((1+a_0 b_0 )q^2 +(a_0 b_1 +a_1 b_0 )q^3 +(a_0 b_2 +a_1 b_1 +a_2 b_0)q^4 + ... =a_0^2 +b_0^2 +2(a_0 a_1 +b_0 b_1 )q+(2a_0 a_2 +a_1^2 +2b_0 b_2 +b_1^2 )q^2 +...\) und im Koeffizientenvergleich zunächst ebenfalls \(a_0^2 +b_0^2 =0\) und damit \(a_0 =b_0 =0\). Aber schon für die nächsten Koeffizienten gilt \(a_1^2 +b_1^2 =1\), also insbesondere (o.B.d.A. wegen der Symmetrie zwischen und a und b) \(a_1^2=1\) und \(b_1=0\). Das aber bedeutet bereits, es könnte Lösungspaare (a,b) geben für die die in der Aufgabenstellung behauptete Eigenschaft zutrifft. Um den Beweis abzuschließen, müssen wir noch prüfen, dass es für \(c=q^l\) mit \(l>2\) keine Lösungen gibt. Wir setzen also \((1+a_0 b_0 )q^l +(a_0 b_1 +a_1 b_0 )q^{l+1} +(a_0 b_2 +a_1 b_1 +a_2 b_0)q^{l+2} + ... =a_0^2 +b_0^2 +2(a_0 a_1 +b_0 b_1 )q+(2a_0 a_2 +a_1^2 +2b_0 b_2 +b_1^2 )q^2 +...\) Hier erhält man, wie man leicht sieht, in Folge stets \(a_k^2 +b_k^2 =0\) und damit \(a_k =b_k =0\) für alle \(k\ge 0\), sprich auch hier erhält man lediglich die trivialen Lösungen a = b = c = 0. Wir können also letztlich sagen, dass, wenn c überhaupt eine natürliche Zahl ist, diese eine Quadratzahl sein muss. q.e.d. Sei (a,b) ein nichttriviales Lösungspaar der Gleichung (1) \(\frac{a^2+b^2}{ab+1}=c\) (also \(c>1\)). Dann lässt sich aus diesem Paar eine Folge \(g(c)=\{g_i, i\in \IZ\}\) ganzer Zahlen erzeugen, für die gilt \(\frac{g_i^2+g_{i+1}^2}{g_i g_{i+1} +1}=c\) (2) mit stets demselben c. Beweis: Sei o.B.d.A. \(a>b\), dann lässt sich ein \(a'>a\) aus \(\frac{a'^2+a^2}{a'a+1}=c\) wie folgt berechnen. Umgestellt ergibt sich die quadratische Gleichung \(a'^2-aca'+a^2-1=0\) für a'. Setzen wir für \(c=\frac{a^2+b^2}{ab+1}\) ein, lässt sich diese quadratische Gleichung elementar lösen mit den Ergebnissen \(a'=b\) (für uns ungeeignet, weil \(a'>a\) sein soll) und \(a'=\frac{a^3-b}{ab+1}\). Dieser Wert ist tatsächlich größer als a und er ist ganzzahlig, wie man nach einer kurzen Umrechnung feststellen kann. Dazu wird im Zähler \(ab^2-ab^2\) "addiert", was \(a'=ac-b\) ergibt. Analog lässt sich ein \(b'a' bzw. b''hier). Deshalb berechnen wir jetzt insbesondere \(g_0\), also den kleinsten nichtnegativen Wert (die \( g_i\) für alle \(i<0\) sind negativ, entsprechend sind die \( g_i\) für alle \(i\ge0\) nichtnegativ). Dies erschliesst sich erstaunlich leicht durch folgende Überlegung: es ist \( g_0=0\). Wäre dies nämlich nicht der Fall, dann gäbe es mit \(g_0\) und \(g_{-1}\) zwei Nachbarn für die die obige Gleichung (2) entgegen ihrer Konstruktion nicht gelten würde. Negative und positive Werte von g(c) liegen also symmetrisch bzgl. der Null. Für \(g_1\) setzen wir nun einfach und vor allem in aller Allgemeinheit q. Die nichtnegativen Werte von g(c) ergeben sich damit als die bereits unten in den Kommentaren von mir angegebenen \( f_k(q)\). Weitere Lösungen gibt es nicht. Und insbesondere ist mit \(\frac{g_1^2+g_0^2}{g_1g_0+1}=q^2=c\) nun auch gezeigt, dass c stets eine Quadratzahl ist. q.e.d.

Bonus

Mit dem Beweis haben wir aber noch mehr gewonnen, nämlich eine Möglichkeit zur Konstruktion von a und b. Damit können wir dann auch zeigen, ob c überhaupt eine natürliche Zahl sein kann (ausser für die trivialen Fälle). Dabei ist folgendes zu berücksichtigen: 1. n und m müssen endlich sein, d.h. sie müssen terminiert werden. Dabei gehen wir so vor, dass wir peu á peu für a das Laufindexende n auf 1, dann auf 2, 3 usw. setzen, sprich es werden dadurch jeweils alle anderen \(a_i =0\) für \(i>n\). Daraus ermitteln wir, wenn möglich, das zugehörige b. 2. Natürlich ist die q-adische Darstellung eindeutig und die Koeffizienten sind die möglichen Reste der Division durch q, also die Ziffern 0, 1, 2, ..., q-1; es wäre also bspw. \((q-1)\cdot q^2\) eine solche Zahl. Da q aber unbekannt ist, könnte dies als \(-q^2 +q^3\) erscheinen, sprich wir lassen auch negative ganzzahlige Koeffizienten zu. Sollte es Lösungen mit negativen ganzen Zahlen geben, dann sind diese leicht in korrekter q-adischer Darstellung angebbar. Daher sind insbesondere zwei Fälle zu beachten nämlich \(a_1=1\) und \(a_1=-1\), in beiden Fällen ist \(b_1=0\). Für \(n=1\) und \(a_1=1\) (also \(a_i=0\) für \(i>1\)) finden wir folgende Gleichungen für die jeweiligen Koeffizienten: \(q^3 : a_0 b_1 +a_1 b_0 =0=2(a_0 a_3 +a_1 a_2 +b_0 b_3 +b_1 b_2 )\), was erfüllt ist. \(q^4 : a_0 b_2 +a_1 b_1 +a_2 b_0 =0=2a_0 a_4 +2a_1 a_3 +a_2^2 +2b_0 b_4 +2b_1 b_3 +b_2^2\), was \(b_2=0\) ergibt. \(q^5\) : erfüllt sich aus dem bisherigen zu 0, ergibt also nichts Neues. \(q^6\) : ergibt \(b_3 = b_3^2\), also \(b_3=1\). Alle anderen \(b_i=0\) für i > 3. D.h. unsere erste Lösung lautet \(a=q\) und \(b=q^3\) für beliebige q (bspw. a=2 und b=8 wäre das erste nichttriviale Beispiel und ergibt 4). Ganz analog erhält man \(a=-q+q^5\) und wiederum \(b=q^3\), hier wäre a=30, b=8 das erste Beispiel.

Schluss

Wir haben nun gezeigt, dass es Lösungen für c in den natürlichen Zahlen gibt. Insbesondere konnten wir zwei Lösungsfamilien finden. Ob es noch mehr Lösungsfamilien gibt, überlasse ich dem Leser. Viel Freude trunx (Jens Koch)
\(\endgroup\)
Get link to this article Get link to this article  Printable version Printer-friendly version -  Choose language     Kommentare zeigen Comments  
pdfFür diesen Artikel gibt es keine pdf-Datei


Write a comment

Arbeitsgruppe Alexandria Dieser Artikel ist nicht im Verzeichnis der Arbeitsgruppe Alexandria eingetragen.
[Die Arbeitsgruppe Alexandria katalogisiert die Artikel auf dem Matheplaneten]
 


 
 
Aufrufzähler 2317
 
Aufrufstatistik des Artikels
Insgesamt 123 externe Seitenaufrufe zwischen 2020.05 und 2023.11 [Anzeigen]
DomainAnzahlProz
http://www.hjcaspar.de86.5%6.5 %
https://google.com4435.8%35.8 %
https://google.de6351.2%51.2 %
https://www.startpage.com64.9%4.9 %
https://yandex.ru10.8%0.8 %
https://eu.startpage.com10.8%0.8 %

Häufige Aufrufer in früheren Monaten
Insgesamt 102 häufige Aufrufer [Anzeigen]
DatumAufrufer-URL
2020-2023 (46x)https://google.de/
2020-2023 (36x)https://google.com/
2020-2022 (14x)https://google.de
2021-2023 (6x)https://www.startpage.com/


[Top of page]



"Mathematik: Ein schwieriges Problem auf der IMO" | 42 Comments
The authors of the comments are responsible for the content.

Re: Ein schwieriges Problem auf der IMO
von: Triceratops am: So. 08. Dezember 2019 10:07:29
\(\begingroup\)"Während für die erste dieser Aufgaben auch eine Lösung verlinkt wurde, habe ich für die zweite Aufgabe keine Lösung im Internet gefunden." Das verwundert mich etwas. Es ist immerhin eine der berühmtesten Aufgaben schlechthin. Ich habe (IMO 1988) in ecosia eingegeben und u. a. folgende Links mit ausführlichen Lösungen gefunden: • https://artofproblemsolving.com/wiki/index.php?title=1988_IMO_Problems/Problem_6 • https://www.emis.de/journals/BAMV/conten/vol18/BAMV_XVIII-2_p143-152.pdf • https://math.stackexchange.com/questions/28438/alternative-proof-that-a2b2-ab1-is-a-square-when-its-an-integer • https://math.stackexchange.com/questions/483771/imo-1988-problem-6 • https://prase.cz/kalva/imo/isoln/isoln886.html • https://www.youtube.com/watch?v=osT2FEp7u8o Das Problem wurde auch bei Numberphile besprochen: • https://www.youtube.com/watch?v=L0Vj_7Y2-xY Bei dem ersten stackexchange-Link wird auch folgende Verallgemeinerung erwähnt und bewiesen: Wenn $\frac{a^n+b^n}{(ab)^{n-1}+1}$ eine natürliche Zahl ist, dann ist es sogar eine $n$-te Potenz.\(\endgroup\)
 

Re: Ein schwieriges Problem auf der IMO
von: zippy am: So. 08. Dezember 2019 10:08:52
\(\begingroup\)Das ist eine hübsche Überlegung, aber ich verstehe nicht, warum du von einem Beweis sprichst. Um in einer Gleichung $\sum_iA_i\,q^i=\sum_iB_i\,q^i$ einen Koeffizientenvergleich machen zu können, müsstest du zuvor sicherstellen, dass die Koeffizienten eine passende Zusatzeigenschaft wie etwa $0\le A_i,B_i\(\endgroup\)
 

Re: Ein schwieriges Problem auf der IMO
von: trunx am: So. 08. Dezember 2019 10:10:24
\(\begingroup\)@triceratops: Danke für die Links. Ich habe auch nicht wirklich gesucht, sondern direkt begonnen zu rechnen. @zippy: dieses Wissen setze ich voraus, wie man bereits wenige Zeilen später im 2. Abschnitt lesen kann. bye trunx\(\endgroup\)
 

Re: Ein schwieriges Problem auf der IMO
von: zippy am: So. 08. Dezember 2019 10:26:09
\(\begingroup\)Sorry, aber ich verstehe nicht, was du mir mit "dieses Wissen setze ich voraus, wie man bereits wenige Zeilen später im 2. Abschnitt lesen kann" sagen willst.\(\endgroup\)
 

Re: Ein schwieriges Problem auf der IMO
von: trunx am: So. 08. Dezember 2019 10:33:50
\(\begingroup\)@zippy: dann versuche ich es anders: per definitionem nehmen in der q-adischen Darstellung einer Zahl die Koeffizienten Werte zwischen 0 und q-1 an. D.h. mit der Verwendung des Begriffs "q-adische Darstellung" und gerade eben nicht Polynom in q (über den ganzen Zahlen) ist die Einschränkung der Koeffizienten implizit, aber dennoch klar gegeben. Aber zunächst wird diese Eigenschaft nicht benötigt, denn die Koeffizienten im Beweis sind entweder 0 oder halt nicht. Der genaue Wert spielt zunächst keine Rolle.\(\endgroup\)
 

Re: Ein schwieriges Problem auf der IMO
von: zippy am: So. 08. Dezember 2019 10:49:49
\(\begingroup\)Annehmen kannst du Eigenschaften der Koeffizienten für die Darstellungen von $a$ und $b$. Einen Koeffizientenvergleich machst du aber in der Gleichung $(1+a_0 b_0)q+(a_0 b_1 +a_1 b_0 )q^2+\cdots=a_0^2 +b_0^2 +2(a_0 a_1 +b_0 b_1 )q+(2a_0 a_2 +a_1^2 +2b_0 b_2 +b_1^2 )q^2 +\cdots$. Du schließt beispielweise auf $a_0^2 +b_0^2=0$, weil $0$ der Koeffizient von $q^0$ auf der linken Seite ist. Dieser Schluss ist aber nicht ohne Weiteres richtig, weil ja nicht $0\le a_0^2 +b_0^2\(\endgroup\)
 

Re: Ein schwieriges Problem auf der IMO
von: trunx am: So. 08. Dezember 2019 11:17:16
\(\begingroup\)Ich verstehe, was du meinst (man kann von \(a_0^2 +b_0^2 =0\) nicht ohne Weiteres auf \(a_0 =b_0 =0\) in den q-adischen Zahlen schliessen, für q=2 wäre zb. 1+1=0). Aber der Einwand ist unerheblich, denn obwohl ich mit den q-adischen Zahlen starte, mache ich den Koeffizientenvergleich wegen der Übertragsfreiheit zwischen zwei Polynomen. Ursprünglich habe ich tatsächlich auch mit Polynomen begonnen, wegen der Uneindeutigkeit in der Darstellung von a und b bin ich aber davon abgegangen. Man könnte also auch sagen, ich habe unter den vielen verschiedenen Polynomdarstellungen von a und b die eindeutige q-adische gewählt. Aus diesem Grund ist dann eben doch der fragliche Schluss möglich.\(\endgroup\)
 

Re: Ein schwieriges Problem auf der IMO
von: zippy am: So. 08. Dezember 2019 11:47:14
\(\begingroup\)Du willst also nicht mehr die die Koffizenten zweier q-adischer Entwicklungen, sondern die zweier Polynome vergleichen. Dafür müsste sich $(1+a_0 b_0)q+(a_0 b_1 +a_1 b_0 )q^2+\cdots=a_0^2 +b_0^2 +2(a_0 a_1 +b_0 b_1 )q+(2a_0 a_2 +a_1^2 +2b_0 b_2 +b_1^2 )q^2 +\cdots$ als Gleichung zwischen zwei Polynomen lesen lassen. Das würde voraussetzen, dass $q$ eine Variable ist. Oder anders formuliert: Dass diese Gleichung für feste Werte von $a_i$ und $b_i$ für hinreichend viele verschiedene Werte von $q$ gilt. Dass ist aber an der Stelle, wo du auf $a_0^2 +b_0^2 =0$ schließt, nicht der Fall, denn diese Gleichung leitest du nur für den einen Wert $q=c$ her.\(\endgroup\)
 

Re: Ein schwieriges Problem auf der IMO
von: trunx am: So. 08. Dezember 2019 11:59:57
\(\begingroup\)nein. q ist zwar eine Unbekannte, aber nicht unbedingt variabel, entscheidend für den Einsatz von q ist, dass es gerade die in der Aufgabenstellung gesuchte Struktur von c widerspiegelt. Es würde nichts bringen, sprich einen Beweis auf diesem Wege unmöglich machen, wenn man darauf verzichtete (und bspw. auch c allgemein q-adisch darstellte).\(\endgroup\)
 

Re: Ein schwieriges Problem auf der IMO
von: zippy am: So. 08. Dezember 2019 12:11:59
\(\begingroup\)Mir ist leider nicht klar, was du mit "q ist zwar eine Unbekannte, aber nicht unbedingt variabel" meinst. Willst du wirklich behaupten, dass man aus einer Gleichung $\sum_iA_i\,q^i=\sum_iB_i\,q^i$, in der die $A_i$ und $B_i$ irgendwelche ganze Zahlen sind und die für genau einen Wert von $q$ gilt, auf $A_i=B_i$ schließen kann?\(\endgroup\)
 

Re: Ein schwieriges Problem auf der IMO
von: Slash am: So. 08. Dezember 2019 14:39:02
\(\begingroup\)Ohne dich jetzt persönlich angreifen zu wollen, trunx, verstehe ich nicht, warum man ein solches Thread-Thema zu einem Artikel verwursten muss. Klar, auf der Frontpage gibt es mehr Aufmerksamkeit, aber gleichzeitig zieht es den guten ersten Eindruck des MP runter. Denn - Hand auf's Herz - es ist schon ziemlich hingeschludert für einen Artikel. Meine Kritik geht ebenso an Matroid, der ja die Artikel freigeben kann oder auch nicht. Gruß, Slash\(\endgroup\)
 

Re: Ein schwieriges Problem auf der IMO
von: trunx am: So. 08. Dezember 2019 15:21:13
\(\begingroup\)@zippy: wieso sollten diese gleichungen nur für genau ein q gelten? in die lösungspaare \((a,b) = (q, q^3)\) kannst du für q alle natürlichen zahlen einsetzen. ich habe die polynomgleichung so allgemein wie möglich gehalten, aber eben auch alles an struktur hinein genommen, was durch die aufgabenstellung gegeben war. das schränkt die allgemeinheit ein, aber ermöglicht auch die lösung. und ja, bei einem koeffizientenvergleich sind die koeffizienten gleich, das ist der regelmäßige sinn eines koeffizientenvergleichs. könntest du genauer erläutern, was dir unklar ist, ich verstehe es nicht.\(\endgroup\)
 

Re: Ein schwieriges Problem auf der IMO
von: trunx am: So. 08. Dezember 2019 15:47:49
\(\begingroup\)@slash: "was ich selber denk und tu, trau ich auch dem andern zu." das zum thema "aufmerksamkeit" heischen und "schludrigkeit". und - hand-aufs-herz - du greifst mich, wie auch andere hier auf dem mp, sehr wohl wie sooft persönlich an. also geh in deine witzecke mit hölzchen spielen.\(\endgroup\)
 

Re: Ein schwieriges Problem auf der IMO
von: Diophant am: So. 08. Dezember 2019 15:57:01
\(\begingroup\)Hallo trunx, zu dem Thema wollte ich auch noch loswerden, dass ich zwar momentan nicht den Kopf habe, die fachliche Debatte um deine Lösung zu verfolgen, aber sehr wohl der Ansicht bin, dass eine Lösung einer solchen Aufgabe einen Artikel wert ist. Er ist auch - selbstredend - bereits in der AG Schulmathematik verlinkt (die sich ja besonders an interessierte und ambitonierte Schülerinnen und Schüler richtet). 😄 Gruß, Diophant\(\endgroup\)
 

Re: Ein schwieriges Problem auf der IMO
von: zippy am: So. 08. Dezember 2019 16:40:17
\(\begingroup\)Zunächst einmal, um nicht falsch verstanden zu werden: Auch ich finde das Thema interessant genug für einen Artikel und würde den vorliegenden Artikel auch nicht als "hingeschludert" bezeichnen. Mir geht es nur darum, dass meiner Ansicht nach der Beweis, so wie er im Augenblick dasteht, falsch ist. \quoteon(trunx) @zippy: wieso sollten diese gleichungen nur für genau ein q gelten? in die lösungspaare (a,b)=(q,q3) kannst du für q alle natürlichen zahlen einsetzen. \quoteoff Ich beziehe micht nur auf den Abschnitt Beweis, der Abschnitt Bonus interessiert mich erstmal nicht. Dort leitest du die Gleichung $(1+a_0 b_0)q+(a_0 b_1 +a_1 b_0 )q^2+\cdots=a_0^2 +b_0^2 +2(a_0 a_1 +b_0 b_1 )q+(2a_0 a_2 +a_1^2 +2b_0 b_2 +b_1^2 )q^2 +\cdots$ her und machst dann einen Koeffizientenvergleich. Du leitest diese Gleichung aber nur für genau ein $q$ her, nämlich für $q=c=\frac{a^2 +b^2}{ab+1}$.\(\endgroup\)
 

Re: Ein schwieriges Problem auf der IMO
von: Kornkreis am: So. 08. Dezember 2019 16:58:44
\(\begingroup\)Hi Trunx, ich sehe hier gleich mehrere Probleme, und ich verstehe nicht ganz, warum du so leichtfertig darüber hinweggehst. Deine Zeile "es wäre also a = b = c = 0" beim ersten Koeffizientenvergleich zeigt doch bereits, dass der Koeffizientvergleich so nicht funktioniert - schließlich hast du ein allgemeines q hergenommen, welches auch eine Quadratzahl sein könnte. Auch dass es reichen könnte, lediglich $c=q^l$ für $l>2$ und nicht alle anderen Möglichkeiten (gemischte Potenzen beispielsweise) auszuschließen, müsste gut begründet werden und wäre doch ein zentraler Schritt für einen Beweis gewesen.\(\endgroup\)
 

Re: Ein schwieriges Problem auf der IMO
von: Triceratops am: So. 08. Dezember 2019 17:33:19
\(\begingroup\)@trunx: Ich stimme Kornkreis und zippy zu. Der Beweis funktioniert leider nicht. Ich sehe auch nicht, wie er zu retten wäre.\(\endgroup\)
 

Re: Ein schwieriges Problem auf der IMO
von: trunx am: So. 08. Dezember 2019 20:07:57
\(\begingroup\)So, ich versuche auf alle Fragen zu antworten. Vorausschicken möchte ich, dass ich über kein Argument leichtfertig hinweg gehe. Vielleicht liegt das Problem darin, dass ich nicht allgemein bei den Polynomen geblieben bin und die q-adischen Zahlen ins Spiel gebracht habe. Ich beginne mal mit Kornkreis, denn genau die Punkte haben mich selbst sehr beschäftigt. Zunächst das Argument, dass q als allgemeine Zahl ja bereits eine Quadratzahl sein könnte, aber natürlich sind auch alle Zahlen der Form \(q^l\) für gerade l stets Quadratzahlen. Warum sollten die ausgeschlossen werden? Ich greife für die Antwort auf zippy's Variablenargument zurück. Wie oben bereits gesagt, habe ich zunächst mit Polynomen begonnen und auch c als Polynom in q angenommen. Die Koeffizienten sollten zunächst beliebige ganze Zahlen sein, q irgendeine natürliche Zahl. Für die Koeffizienten \(a_0\), \(b_0\), und \(c_0\) gilt dann insbesondere die Ausgangsgleichung. Da das wenig zielführend war, habe ich zunächst c reduziert, sprich ich habe geschaut, ob es Polynome a(q) und b(q) derart gibt, dass sie in der Aufgabenkonstellation einfach nur q ergeben. Das ist wie oben gezeigt nicht möglich. Für \(c=q^2\) ist es möglich, für \(c=q^l\) und \(l>2\) wieder nicht. Der Grund für die Einführung der q-adischen Zahlen war, dass ich Polynome eher als Funktionen sehe. Damit habe ich mir nun offenbar mehr Probleme ins Haus geholt, als mir lieb war. Da ich im Beweis aber irgendwo wirklich die Eigenschaften q-adischer Zahlen nutze, selbst im Bonusteil nicht, kann man darauf wohl verzichten. Dass ich für c reine und keine gemischten Potenzen angesetzt habe, liegt daran, dass in den gemischten Potenzen c letztlich wieder einfach nur als normale unstrukturierte Zahl vorliegt und dieser Fall ist ja bereits mit \(c=q\) erledigt. Höhere Potenzen hingegen mussten logischerweise berücksichtigt werden, denn deren Struktur ist nicht durch durch c=q abgedeckt.\(\endgroup\)
 

Re: Ein schwieriges Problem auf der IMO
von: trunx am: So. 08. Dezember 2019 21:21:45
\(\begingroup\)Ich versuche, diesen Gedankengang mal mit einem Beispiel zu illustrieren: Eine Lösung ist ja c=4 mit dem Lösungspaar (2, 8). Wenn ich jetzt q=4 setze, dann kann ich ja Polynome a(q) und b(q) in q bilden, für die insbesondere a(4)=2 und b(4)=8 gilt. Bspw. die 4adische Darstellung von 2 und 8, also \(a(4)=2\cdot 4^0\) und \(b=2\cdot 4^1\). Für das Problem der Aufgabenstellung sollten nun aber für jedes q die Paare \((a(q)=2, b(q)=2q)\) Lösungspaare sein und q ergeben. Setzen wir nun aber in diese Paare q=5 ein, erhalten wir aber statt des geforderten c=5 nicht mal eine natürliche Zahl, sondern \(c=\frac{104}{21}\). Was ich oben also gezeigt habe ist, dass es keine Polynompaare (a(q), b(q)) gibt, die für alle natürlichen Zahlen q sowohl Lösungspaare der Aufgabenstellung als auch im Ergebnis q sind. Es geht mir also gerade nicht um ein spezielles q, sondern betrifft stets alle natürlichen Zahlen. Jetzt könnte noch man argumentieren, dass es ja unter allen Lösungen eine einzige gibt, die keine Quadratzahl ist, aber eben aufgrund der Einzigkeit nicht von Lösungspolynomen erfasst werden würde. Aber auch das ist nicht möglich, denn dann müsste der Koeffizientenvergleich irgendwo nicht möglich sein und die Gleichung insgesamt nur durch das spezielle q zu retten sein. Doch das lieferte nur triviale Lösungen.\(\endgroup\)
 

Re: Ein schwieriges Problem auf der IMO
von: Slash am: So. 08. Dezember 2019 21:23:28
\(\begingroup\)trunx @slash: "was ich selber denk und tu, trau ich auch dem andern zu." das zum thema "aufmerksamkeit heischen" und "schludrigkeit". und - hand-aufs-herz - du greifst mich, wie auch andere hier auf dem mp, sehr wohl wie sooft persönlich an. also geh in deine witzecke mit hölzchen spielen. "aufmerksamkeit heischen": das sind deine Worte, nicht meine! "du greifst mich, wie auch andere hier auf dem mp, sehr wohl wie sooft persönlich an": Ich wüsste nicht, wo ich dich in der Vergangenheit schon mal persönlich angegriffen hätte? Beweise? Und wenn dies anderen tun, kann ich nun wirklich nichts dafür. Wenn man aber im Wochenrhythmus langatmiges unfachliches Gefasel (Artikel) zu Themen wie Zeit und Urknall veröffentlicht (Geltungsbedürfnis), dann darf man sich nicht über (scharfe) Kritik wundern. Nicht jeder ist an deinen perönlichen Gedanken interessiert. aus deinem Artikel: "Während für die erste dieser Aufgaben auch eine Lösung verlinkt wurde, habe ich für die zweite Aufgabe keine Lösung im Internet gefunden." trunx @triceratops: Danke für die Links. Ich habe auch nicht wirklich gesucht, sondern direkt begonnen zu rechnen. ...und das ist nicht als schludrig zu bezeichnen? 😵 Und zu deinen vermeintlichen Beweis: Hättest du jemand anderen Probelesen lassen, hättest du diesen Beweis direkt nochmal überarbeiten können. ...nein, das ist kein schludriges arbeiten. 😛 ...und wenn dein Beweis falsch "bleibt", dann hat dein Artikel genauso schnell wieder zu verschwinden, wie er aufgetaucht ist - das versteht sich hoffentlich von selbst. In einem neuen Thread darf dann gerne weiter diskutiert werden. Dagegen spricht ja nichts.\(\endgroup\)
 

Re: Ein schwieriges Problem auf der IMO
von: zippy am: So. 08. Dezember 2019 21:54:29
\(\begingroup\)\quoteon(trunx) sprich ich habe geschaut, ob es Polynome a(q) und b(q) derart gibt, dass sie in der Aufgabenkonstellation einfach nur q ergeben. Das ist wie oben gezeigt nicht möglich. \quoteoff Du hast tatsächlich gezeigt, dass für Polynome $a(q)$ und $b(q)$ die Gleichung$$ q\cdot\bigl(a(q)\,b(q)+1\bigr)=a(q)^2 +b(q)^2 $$nicht als Gleichung zwischen Polynomen gelten kann. Dass es dir wirklich um diese Polynomgleichung geht, bestätigst du einen Kommentar später auch nochmal: \quoteon(trunx) Was ich oben also gezeigt habe ist, dass es keine Polynompaare (a(q), b(q)) gibt, die für alle natürlichen Zahlen q sowohl Lösungspaare der Aufgabenstellung als auch im Ergebnis q sind. \quoteoff Das Problem ist aber, dass diese Polynomgleichung stärker ist als die tatsächlich zu widerlegende Aussage. Wenn dir das nicht klar ist, schreib doch mal deine Herleitung dieser Polynomgleichung aus der Aufgabenstellung auf und verzichte dabei auf jegliches Hand-waving.\(\endgroup\)
 

Re: Ein schwieriges Problem auf der IMO
von: trunx am: Mo. 09. Dezember 2019 16:10:11
\(\begingroup\)Ich habe heute mit einem Kollegen über das Problem gesprochen und verstehe nun die Schwierigkeiten mit dem obigen Text besser. Demnach ist es zwar immer ganz nett, wenn man einige (mitunter auch unendlich viele) Lösungen berechnen kann, aber das ist noch keine Aussage über alle Lösungen (klar). Auch er hält den Text für keinen Beweis, sondern lediglich für einen Ansatz, den man deshalb auch nicht retten, sondern allenfalls zu Ende führen könne. Dazu wäre zumindest zu zeigen, dass Polynompaare (a(q), b(q)), die für alle \(q \in \mathbb{N}\) Lösungen der obigen Gleichung liefern, zugleich auch die einzige Möglichkeit für solche Lösungen darstellen, wenn es denn überhaupt so ist. Entsprechend wären dann nur solche Polynome c(q) Lösungspolynome, wenn sie wiederum für alle \(q \in \mathbb{N}\) Lösungen liefern. Wenn bestimmte c(q) nur an einzelnen Stellen Lösungswerte annehmen und an anderen nicht, gehören diese Lösungswerte zu einem anderen Lösungspaar. Desweiteren sind für ihn alle Quadrate von Polynomen mögliche Kandidaten für Lösungspolynome c(q), insbesondere auch mit gemischten Potenzen. Warum ich hier was ausschliesse, müsste ich sehr genau begründen usw. Wie auch immer, ich habe offenbar nur eine Möglichkeit gefunden, ein paar Lösungen zu berechnen (und wer weiss, vielleicht sogar alle), aber eben keinen Beweis. Schade. Ich danke allen, die mir zu dieser Erkenntnis auf die Sprünge geholfen haben. bye trunx \(\endgroup\)
 

Re: Ein schwieriges Problem auf der IMO
von: Gerhardus am: Di. 10. Dezember 2019 12:16:02
\(\begingroup\)Schöne Aufgabe, passt optimal in meinen nächsten Artikel. Der IMO-Beweis ergibt: Die Tupel (a,b,c) = (n,n³,n²) lösen die eingangs gestellte Gleichung. a und b sind vertauschbar. Die Lösung (8,30,4) ist von anderer Struktur. \(\endgroup\)
 

Re: Ein schwieriges Problem auf der IMO
von: trunx am: Di. 10. Dezember 2019 17:27:07
\(\begingroup\)@gerhardus: wie oben unter "Bonus" geschrieben, hat (8,30,4) in der Tat eine andere Struktur und gehört zu den Lösungspolynomen \((q^3,-q+q^5,q^2)\), hier für q=2. Allgemeiner kann man eine Folge von Polynomen, ich nenne sie mal \(f_n(q)\) angeben, von denen je zwei benachbarte ein Lösungspaar (a(q),b(q)) sind, also \(a(q)=f_n(q)\) und \(b(q)=f_{n+1}(q)\). Die ersten \(f_n(q)\) lauten: \(0,q,q^3,-q+q^5,-2q^3+q^7,q-3q^5+q^9,...\) Für die \(f_n(q)\) gilt folgende Rekursion: \(f_{n+1}(q)\cdot f_{n-1}(q)=f_n^2(q)-q^2\) oder können allgemein angegeben werden \[f_n(q)=\sum \limits_{i=0}^{\lceil\frac{n}{2}\rceil -1} (-)^i \binom{n-i-1}{i} q^{2n-4i-1}\]\(\endgroup\)
 

+
von: haegar90 am: Sa. 14. Dezember 2019 12:39:50
\(\begingroup\)\(\endgroup\)
 

Re: Ein schwieriges Problem auf der IMO
von: zippy am: Sa. 14. Dezember 2019 15:27:07
\(\begingroup\)\quoteon(trunx) Die nachgelieferte Zuendeführung des angekündigten Beweises findet sich im nächsten Abschnitt in blauer Schrift. \quoteoff Das ist weniger eine "Zuendeführung" als ein komplett neuer Beweis (und zwar der aus Triceratops' drittletztem Link).\(\endgroup\)
 

Re: Ein schwieriges Problem auf der IMO
von: trunx am: Sa. 14. Dezember 2019 17:34:07
\(\begingroup\)@zippy: das mag bei einem alten, vielfach gelösten Problem der Fall sein. Das Programm, dass ich mir aber zum Beweisen für dieses WE vorgenommen hatte, steht in meinem Kommentar vom 9.12. und das habe ich so auch fortgeführt. Das direkt am Anfang dieses Wegs nun gleich ein Beweis lag, den ich bemerkte (wahrscheinlich lagen mir noch andere, aber eben unbemerkt gebliebene Beweise vor der Nase), ändert daran nichts, hat aber gleichzeitig den Rest des Programms überflüssig gemacht. Überdies benutzte ich bei genauerer Betrachtung für meinen obigen Beweis Ergebnisse aus dem ersten Beweisversuch, insbesondere die letztlich entscheidende Eigenschaft der Lösungen, rekursiv ermittelbar zu sein, die mich ja auch zu den Polynomen geführt hatte, weshalb ich dort logischerweise auch ansetzte.\(\endgroup\)
 

Re: Ein schwieriges Problem auf der IMO
von: weird am: Sa. 14. Dezember 2019 17:40:54
\(\begingroup\)@zippy: Ja, man muss das jetzt in der letzten Version leider als den unredlichen Versuch ansehen, so zu tun, als ob an der ursprünglichen Beweisidee noch irgendetwas etwas zu retten gewesen wäre. Schade eigentlich, denn vorher hatte es noch so ausgesehen, als hätte er seinen Fehler ohne Wenn und Aber eingesehen. ☹️ \(\endgroup\)
 

Re: Ein schwieriges Problem auf der IMO
von: Hans-Juergen am: Sa. 14. Dezember 2019 18:44:06
\(\begingroup\) Hi, in dem Bruch (a^2+b^2)/(1+a b) sind zwei natürliche Zahlen a und b so zu bestimmen, dass er gleich dem Quadrat einer ebenfalls natürlichen Zahl n wird. Es soll also gelten: (a^2+b^2)/(1+a b)= n^2 mit a,b,n\el\ \IN . Ich setze a = x n, b = y n ; dann folgt: (x^2 n^2 + y^2 n^2)/(1 + x y n^2) = n^2 <=> (x^2 + y^2)/(1 + x y n^2) = 1 <=> x^2 + y^2 = 1 + x y n^2 , y^2 - x n^2 y + x^2 - 1 = 0. => y = (x n^2 +- sqrt(x^2 n^4 - 4(x^2 - 1)))/2 . Mit dem oberen Vorzeichen ergibt sich für x=1 y=n^2, somit a=n, b=n^3 , d. h. es ist (a^2 + b^2)/(1 + a b) = (n^2 + nx^6)/(1+ n^4) = (1+n^4)/(1+n^4)*n^2 = n^2 wie gefordert. a=n, b=n^3 entspricht dem von Gerhardus am Di. um 12:16:02 genannten IMO-Beweis, den ich nicht kenne. \(\endgroup\)
 

Re: Ein schwieriges Problem auf der IMO
von: trunx am: Sa. 14. Dezember 2019 20:01:29
\(\begingroup\)hallo Hans-Jürgen, deine Rechnung ist leider auch kein Beweis und berücksichtigt deutlich weniger Lösungen als ich es in meinem obigen Beweis(ansatz) getan hatte. Auch dort finden sich bereits die Lösungspolynome \((q, q^3)\), und auch die Lösungspolynome \((q^3, q^5-q)\) usw. In meiner Antwort auf gerhardus habe ich darüber hinaus alle Lösungspolynome in allgemeiner Form angegeben. @weird: ehrenrührig zu werden sieht dir nicht ähnlich. Meinen Fehler habe ich eingesehen, nämlich, auch wenn ich das dachte, nicht wirklich fertig geworden zu sein. Das bedeutet aber für mich, mich halt noch mal ranzusetzen und weiter zu machen und zu Ende zu bringen, was ich angefangen habe. Machst du das nicht? Die komplette Lösungsmenge, von der ich mal annehme, dass ihr mir zubilligt, sie oben selbständig ermittelt zu haben, ist ja zweidimensional - es gibt einmal den Laufindex der linearen Rekursion und zum 2. die Variable q in den jeweiligen Lösungspolynomen. Der Plan war nun, wie gesagt, mir anzusehen, ob jede Lösung (a, b) mindestens zu einem Polynompaar (a(q), b(q)), das für alle q aus den natürlichen Zahlen die Problemgleichung löst, gehören muss. Dazu musste ich erstmal in aller Allgemeinheit diese Polynompaare erzeugen und letztlich entstehen sie halt durch die Rekursion. Da die Rekursion auf zwei Vorgänger zurückgreift, sind zwei Startwerte erforderlich um das allgemeine Glied darzustellen. Dieses allgemeine Glied der Rekursion ist mit zwei Startwerten aber eben eine Funktion in zwei Variablen, konkret eine gebrochen-rationale Funktion in zwei Variablen. Um sich hier die Arbeit zu vereinfachen, lag es nahe, und war ohnehin von vornherein so vorgesehen (weil ich ja zu den Polynomen wollte) die Zahl der Variablen um eins zu verringern und das geht nun mal nur auf eine einzige Art und Weise, nämlich in der Ermittlung von \(g_0\). Das habe ich getan und dann war halt schon alles vorbei. Bis dahin habe ich geglaubt, es gäbe wirklich noch weitere Lösungspolynome.\(\endgroup\)
 

Re: Ein schwieriges Problem auf der IMO
von: weird am: Sa. 14. Dezember 2019 23:35:49
\(\begingroup\)@trunx "Ehrenrührig" finde ich meinen obigen Kommentar jetzt wirklich nicht, aber vielleicht doch noch der Klarheit halber nachstehende Präzisierung: Ich fand und finde es noch immer toll, dass du deinen Fehler in der ursprünglichen Beweisführung doch noch eingesehen und ursprünglich dann auch einen entsprechenden Vermerk auch in dem Artikel angebracht hast. Dieser ist nun leider verschwunden und statt dessen hast du einen anderen Ansatz ins Spiel gebracht, der, wie bereits von zippy erwähnt, so verstanden (oder missverstanden?) werden kann, dass man deinen ursprünglichen Beweis doch noch zu einem guten Ende führen kann, was aber beim besten Willen nicht den Tatsachen entspricht. \(\endgroup\)
 

Re: Ein schwieriges Problem auf der IMO
von: trunx am: So. 15. Dezember 2019 01:09:51
\(\begingroup\)@weird: mittlerweile weiss ich soviel über die Fragestellung, dass ich überall Beweise sehe. Und dass ich soviel darüber weiss, hängt wesentlich von meinem Ansatz und meinen eigenen Rechnungen ab und nicht von irgendeinem unterstellten abgekupferten Ansatz. Auch das zur Klarstellung. Nehmen wir z.B. an, ich hätte die Symmetrie in den ganzzahligen \(g_i\) übersehen oder vielleicht sogar, dass die \(g_i\) sogar ganzzahlig sein können. Dann sind die \(g_i\) wie bereits gesagt, gebrochen-rationale Funktionen in den zwei Startwerten der zugehörigen Rekursion \(g_0\) und \(g_1\), aber mit Werten in den natürlichen Zahlen. Dies erkennt man daran, dass diese Funktionen mit c als Polynome geschrieben werden können, nämlich \(g_i= g_1 h_i(c) - g_0 h_{i-1}(c)\), wobei die \(h_i(c)\) ihrerseits rekursiv gebildet werden mit \(h_{i+1}=c h_i - h_{i-1}\) und entsprechend 0, 1, c, \(c^2-1\), \(c^3-2c\),... lauten. Diese \(g_i\) stellen die Verallgemeinerung der oben in meinem Ansatz eingeführten Polynome (a(q), b(q)) dar, was hoffentlich niemand abstreitet. Wenden wir uns den Lösungspolynomen c(q) zu. Es wurde behauptet, dass man nicht wissen könne, wie diese aussehen, insbesondere könnten sie letztlich (schon die Richtigkeit der Aufgabenstellung vorausgesetzt) alle Quadrate von Polynomen sein. Das mag so sein, aber in der Verallgemeinerung gibt es nur eine einzige Funktion für c in \(g_0\) und \(g_1\) und die ist \(\frac{g_0^2 +g_1^2}{g_0 g_1 +1}\) (setzt man höhere Werte für \(g_i\) kürzt sich alles bis auf diesen Rest weg). Damit scheint nicht viel gewonnen, aber schreiben wir diesen Bruch in \(g_1\)-adischer Form: Dazu verwenden wir \(g_0^2=xg_1 +y\), wobei x und y nur Werte zwischen 0 und \(g_1 -1\) in den natürlichen Zahlen annehmen dürfen. Genauer ist sogar \(x0\) sei, folgende Gleichung gelten \(1xy=g_0 1\cdot y\) (der zweite Faktor kann \(g_1\)-adisch nicht zweistellig sein, denn dann müsste \(g_0=1\) und damit 101=11•11 sein, was nicht stimmt, außer für \(g_1=2\) (hier ist 11•11=1001), kommt für 11•11 stets 121 in jedem System heraus). Das aber ergibt folgende Gleichung \(\frac{g_0^2+x^2}{g_0 x+1}=y\). Wäre jetzt x=0, dann wäre \(y=g_0^2\) und damit \(g_1=g_0 y=g_0^3\), was der Minimalität der Startwerte \(g_0\) und \(g_1\) für die Folge g widerspricht, denn es gäbe ein \(g_{-1}=0\), das kleiner wäre. Also ist x>0 und die Gleichung \(\frac{g_0^2+x^2}{g_0 x+1}=y\) ist wegen \(x0\) und spätestens jetzt ist auch mit meinem Ansatz \(g_0=0\) und \(c=g_1^2=q^2\) eine Quadratzahl.\(\endgroup\)
 

Re: Ein schwieriges Problem auf der IMO
von: Hans-Juergen am: So. 15. Dezember 2019 13:45:27
\(\begingroup\)Hallo trunx, Du hast recht: es ist kein allgemeiner Beweis, den ich auch gar nicht anstrebte. Mich interessierte lediglich, wie man auf das von Gerhardus angegebene Tripel (n,n³,n²) kommt. (Schneller geht es übrigens mit dem Ansatz a=nx, b=y.) Mit freundlichem Gruß Hans-Jürgen\(\endgroup\)
 

Re: Ein schwieriges Problem auf der IMO
von: Ex_Mitglied_maxbauer am: So. 15. Dezember 2019 19:11:08
\(\begingroup\)Hallo, der IMO-Beweis ergab a² = c und ca = b, daraus folgen die Lösungen (n,n³,n²), die man durch Einsetzen in die Gleichung unmittelbar verifizieren kann. Weitere Lösungen finden sich mit dem Vieta jumping (Vieta-Sprung-Technik), von dem hier seltsamer Weise noch keiner gesprochen hat, das aber in den im 1. Kommentar gelisteten Links erwähnt wird. Mehr dazu hier. Dort wird die Aufgabe auch besprochen. Gruß, Gerhardus \(\endgroup\)
 

Re: Ein schwieriges Problem auf der IMO
von: Gerhardus am: Di. 17. Dezember 2019 08:49:15
\(\begingroup\)Die bisherigen Threads zum Vieta jumping ließen in ihrer Kürze nicht erkennen, ob die Methode richtig verstanden worden ist. Für Schlaufüchse ist sie auch viel zu simpel. Ich versuche es auf dem Niveau des 9. Schuljahres zu erklären, das mit dem Satz von Vieta vertraut ist. In der Notation von trunx sind (a,b,c) = (q, q³,q²) Lösungen der Gleichung (G1) a² + b² – c∙a∙b – c = 0, weil die Gleichung stimmt, wenn darin a durch q, b durch q³ und c durch q² ersetzt werden. Nun halten wir c = q² fest und suchen nach weiteren Lösungspaaren (a,b), beginnend mit (q,q³). (Dass (q³,q) die symmetrische Gleichung auch löst, reicht uns nicht.) Dazu ersetzen wir die kleinere Lösungszahl a durch x, die Konstante c durch q² und erhalten eine klassische quadratische Gleichung (Vx) x² – q²∙b∙x + b² – q² = 0, die wir mit dem Satz von Vieta lösen, weil eine Lösung a = q bereits bekannt ist. Für die Zusatzlösung a’ gilt dann nach dem Satz von Vieta (V1) a’∙a = b² – q² und (V2) a’ = q²∙b – a Das Lösungspaar (a,b) = (q,q³) ergibt in (V1) und (V2) a’ = q⁵– q. Also ein neues Lösungspaar (a’,b) = (q⁵– q,q³). (V2) garantiert die Fortsetzung der Ganzzahligkeit. Um mit diesem Lösungspaar das Vieta-Spielchen zu wiederholen, vertauschen wir in den Gleichungen (Vx), (V1) und (V2) a und b, und ersetzen a jetzt durch die neue Zahl a’ = q⁵– q: (Vx’) x² – q²∙a’∙x + a’² – q² = 0, (V1’) b’∙b = a’² – q² und (V2’) b’ = q²∙a’ – b. Resultat b’ = q⁷ – 2q³. Neues Lösungspaar (a’,b’) = (q⁵– q, q⁷– 2q³). So wie aufsteigend, funktioniert es auch absteigend und so geht auch der Beweis der Aufgabe zu zeigen, dass c eine Quadratzahl sein muss. Man geht von einem minimalen Lösungspaar (a,b) aus mit 1 ≤ b ≤ a. Die Gleichung (Vx) führt dann zur Lösung a' = 0, die in (G1) die Variable a ersetzt und c = b² bedeutet. Die Regel (V1) steht in der Antwort von trunx auf meinen 1. Kommentar in der Form a = (n–1)-te Polynomfunktion von q, a’ = (n+1)-te Polynomfunktion von q, b = fₙ(q) = n-te Polynomfunktion von q. Indiziert lassen (V1) und (V2) die Folge der Lösungszahlen weiter wachsen. \(\endgroup\)
 

Re: Ein schwieriges Problem auf der IMO
von: trunx am: Di. 17. Dezember 2019 23:03:07
\(\begingroup\)hallo gerhardus, danke für die ergänzung. ich hatte die bei dir V1 genannte rekursionsgleichung empirisch gefunden, dann aber die deutlich leichtere V2 (die natürlich über die problemgleichung zusammen hängen) und daher nur noch letztere verwendet. für den beweis, dass c quadratzahl ist, reicht V1 (oder der vieta-jump) allein nicht aus, vorher ist es zwingend notwendig zu zeigen, dass der kleinste nichtnegative (start)wert der zugehörigen rekursion 0 ist oder halt die rekursion ganzzahlig betrachtet wird und deren werte symmetrisch bzgl. 0 liegen müssen. daher kam diese gleichung auch hier nicht zum einsatz. allerdings gebe ich gern zu, dass ich den satz von vieta hier nicht gesehen hätte. bye trunx\(\endgroup\)
 

Re: Ein schwieriges Problem auf der IMO
von: MartinN am: Di. 17. Dezember 2019 23:19:01
\(\begingroup\)Ich hab grad das dazu gesehen... https://youtu.be/Y30VF3cSIYQ [und deren 2. Teil] Wiki erklärt es mMn auch recht gut: https://en.wikipedia.org/wiki/Vieta_jumping [da wird das Problem ja auch mittels Widerspruchsbeweis gelöst]\(\endgroup\)
 

Re: Ein schwieriges Problem auf der IMO
von: Gerhardus am: Mi. 18. Dezember 2019 20:04:26
\(\begingroup\)Die erste deutschsprachige Darstellung des Vieta jumping habe ich gefunden im Buch Franz Lemmermeyer, Mathematik à la Carte, Quadratische Gleichungen mit Schnitten von Kegeln, 2016, Seite 177 bis 178. Er nennt es Vietas Wurzelwechsel. Uni-Bibliotheken bieten es als E-Book an.\(\endgroup\)
 

Re: Ein schwieriges Problem auf der IMO
von: Hans-Juergen am: Mi. 18. Dezember 2019 21:22:26
\(\begingroup\) Guten Tag Das "Vieta Jumping" ist ein nicht naheliegendes, kaum bekanntes Verfahren. Angewendet und erklärt wurde es bei der vorliegenden Olympiadeaufgabe. Man kann sie aber auch wie folgt ganz einfach ohne VJ lösen. Der Bruch (a^2+b^2)/(1+ab) soll eine Quadratzahl sein; gesucht sind Zahlen a,b\el\ \IN, die das bewirken: (a^2+b^2)/(1+ab)=n^2, n\el\ \IN. a und b hängen irgendwie mit n zusammen, und der einfachste Lösungsansatz ist a=xn, b=yn. Dies ergibt x^2 n^2 + y^2 n^2 = n^2 + n^2 a b . (1) Man |r ä t| leicht: x=1 -> y=n^2 und hat somit a=n, b=n^3. Wir setzen b in (1) ein, um eine weitere Lösungszahl a zu erhalten: a^2 + n^6 =n^2 + n^5 a a^2-n^5 a + n^6 - n^2 =0 a = (n^5 +- sqrt(n^10 - 4(n^6-n^2)))/2 = (n^5 +- n sqrt(n^8 - 4n^4 + 4))/2 = (n^5 +- n(n^4-2))/2. Es gelte das Pluszeichen; dann folgt a=n^5-n, und mit dem vorausgesetzten Term für b ist dies ein weiteres Lösungspaar: (n^5-n,n^3), wie in dem thread. (Anmerkung: würden wir a=n in (1) einsetzen, ergäbe sich b=n^3, was wir schon kennen.) Durch Einsetzen von a=n^5-n in (1) erhält man b = n^7 - 2n^3, wie ebenfalls in dem thread angegeben. Sinngemäß lässt sich das beliebig fortsetzen. Gruß, Hans-Jürgen \(\endgroup\)
 

Re: Ein schwieriges Problem auf der IMO
von: trunx am: Do. 19. Dezember 2019 09:12:39
\(\begingroup\)Hallo Hans-Jürgen, schön, dass dich das Thema interessiert. Was ich aber oben im Artikel gezeigt habe ist, dass man Lösungen nicht raten muss, sondern mit einem naheliegenden, polynomialen Ansatz für a und b (als deren innerer Struktur) und einem Koeffizientenvergleich berechnen kann. Von daher ist dein Ansatz für a nur ein Spezialfall von \(a(q)=a_0+a_1q+...\) viele Grüsse trunx\(\endgroup\)
 

Re: Ein schwieriges Problem auf der IMO
von: Hans-Juergen am: Do. 19. Dezember 2019 14:11:05
\(\begingroup\)Hallo trunx, ich verstehe Dich schon. Wir betrachten das Problem aus verschiedenen Seiten: Du als Fachmathematiker mit dem Anspruch, es möglichst allgemein zu lösen, und ich mit den Augen eines ehemaligen Mathelehrers, speziell im Hinblick auf die 9. Klasse, wie von Gerhardus angedacht. Mein sozusagen konventionelles Vorgehen ist gedanklich einfacher als das meiste, was in diesem thread vorgeschlagen und getan wird; dafür ist bei mir der Schreibaufwand größer als etwa beim Vieta Jumping. Gründlich geübt wird der Umgang mit der Lösungsformel für quadratische Gleichungen und einer der binomischen Formeln. Der obige geratene Ansatz x=1 ist nichts Besonderes, sondern sehr naheliegend, weil durch ihn sich n² auf beiden Seiten heraushebt; außer an dieser Stelle braucht man nirgends zu raten. Frohe Weihnachten! Hans-Jürgen \(\endgroup\)
 

Re: Ein schwieriges Problem auf der IMO
von: trunx am: Do. 19. Dezember 2019 19:58:58
\(\begingroup\)hallo Hans-Jürgen, ich bin kein Fachmathematiker, sondern habe Physik studiert, arbeite aber u.a. als Mathelehrer. Auch dir frohe Weihnacht! trunx\(\endgroup\)
 

 
All logos and trademarks in this site are property of their respective owner. The comments are property of their posters, all the rest © 2001-2023 by Matroids Matheplanet
This web site was originally made with PHP-Nuke, a former web portal system written in PHP that seems no longer to be maintained nor supported. PHP-Nuke is Free Software released under the GNU/GPL license.
Ich distanziere mich von rechtswidrigen oder anstößigen Inhalten, die sich trotz aufmerksamer Prüfung hinter hier verwendeten Links verbergen mögen.
Lesen Sie die Nutzungsbedingungen, die Distanzierung, die Datenschutzerklärung und das Impressum.
[Seitenanfang]